How do I determine orthogonal eigenvectors for a repeated eigenvalue?

Click For Summary
To determine orthogonal eigenvectors for a repeated eigenvalue, it is essential to recognize that while eigenvectors corresponding to distinct eigenvalues of a Hermitian operator are orthogonal, those corresponding to the same eigenvalue are not guaranteed to be. The characteristic equation yields a repeated eigenvalue of 1, leading to a system of equations that simplifies to a single equation, a + b + c = 0. This indicates that there are infinitely many solutions, and thus, one must select specific eigenvectors to ensure orthogonality. The discussion emphasizes the need to choose linearly independent vectors from the two-dimensional subspace spanned by the eigenvectors to achieve orthogonality. Ultimately, careful selection of eigenvectors is necessary to satisfy the orthogonality condition.
J.Asher
Messages
12
Reaction score
0

Homework Statement


There is an Hamiltonian operation which is given by
(2 1 1)
(1 2 1) = H ; 3-by-3 matrix
(1 1 2)
And let's have an arbtrary eigenvector
(a)
(b) = v ; (3x1) matrix
(c)

Then, from the characteristic equation, the eigenvalues are 1,4. Here eigenvalue 1 is
repeated one.

Homework Equations


Now, my question arises. I know that the eigenvectors that corresponds to eigenvalue 1 is two and both are orthgonal to each other. However, I can't find any of them because when
I substitute eigenvalue into 1, I get a kind of meaningless(?) equation,

(2 1 1) (a) (a)
(1 2 1) (b) = (b)
(1 1 2) (c) (c) .

And it gives 3-equivalent equation, a+b+c = 0
I couldn't determine any relation between a,b,c.


The Attempt at a Solution



So I quess that do I have to choose a,b,c satisfying the condition a+b+c=0
but still problem doesn't disappear beacuse there would be a lot of soulutions...

What did I wrong?
 
Last edited:
Physics news on Phys.org
You didn't take it far enough.

a + b + c = 0 ==>
a = -b - c
b = b
c = ... c

The eigenvectors form a two-dimensional subspace of R3 that is spanned by two vectors. If you look closely at the system above, you can pick out the two vectors. These two vectors are linearly independent but they don't happen to be orthogonal.
 
The operator we handle is Hermitian since Hamlitonian is real physical operator.
it is no doubt that the eigenvectors of the Hermitian is orthogonal.
Actually I cannot understand what you are saying. What do you mean by that?
...??
 
J.Asher said:
The operator we handle is Hermitian since Hamlitonian is real physical operator.
it is no doubt that the eigenvectors of the Hermitian is orthogonal.
Actually I cannot understand what you are saying. What do you mean by that?
...??

If it's hermitian the eigenvectors belonging to distinct eigenvalues are orthogonal. Eigenvectors belonging to the same eigenvalue don't have to be. You have to select them to be orthogonal.
 
Question: A clock's minute hand has length 4 and its hour hand has length 3. What is the distance between the tips at the moment when it is increasing most rapidly?(Putnam Exam Question) Answer: Making assumption that both the hands moves at constant angular velocities, the answer is ## \sqrt{7} .## But don't you think this assumption is somewhat doubtful and wrong?

Similar threads

Replies
5
Views
2K
  • · Replies 2 ·
Replies
2
Views
2K
Replies
2
Views
2K
  • · Replies 8 ·
Replies
8
Views
2K
  • · Replies 19 ·
Replies
19
Views
4K
  • · Replies 6 ·
Replies
6
Views
2K
  • · Replies 7 ·
Replies
7
Views
2K
  • · Replies 5 ·
Replies
5
Views
2K
Replies
9
Views
2K
  • · Replies 4 ·
Replies
4
Views
3K